Как найти работу силы ампера в однородном

Как и любая другая сила, сила Ампера имеет возможность совершить работу. По определению механической работы:

displaystyle A=FDelta rcos alpha (1)

  • где

Работа силы Ампера

Рис. 1. Работа силы Ампера

Пусть в нашей системе проводник длиной displaystyle B, находящийся в однородном магнитном поле индукции displaystyle I, по которому течёт ток displaystyle Delta r, движется под действием силы Ампера и перемещается на расстояние displaystyle {{F}_{A}}=IBl (рис. 1). Тогда, при условии, что сила Ампера равна displaystyle {{F}_{A}}=IBl, получим:

displaystyle A=IBlDelta rcos alpha (2)

Пометим displaystyle S=lDelta r — площадь, «заметаемая» при движении проводника. Т.е. площадь, которую «прошёл» проводник во время движения. Тогда, в общем случае:

displaystyle A=IBScos alpha (3)

  • где

Соотношение (3) указывает на работу сил Ампера. Однако, если использовать определение изменения потока магнитного поля:

displaystyle =BDelta Scos alpha Фdisplaystyle =BDelta Scos alpha (4)

получим:

displaystyle A=IDelta Ф (5)

  • где

Работ сил Ампера A = IΔΦ. Здесь ΔΦ имеет смысл модуля магнитного потока сквозь поверхность, заметённую проводником с постоянным током I при его перемещении: ΔΦ = |Φзам|. Знак работы определяется по направлению движения проводника (см. пример 8.1). ΔΦ имеет также смысл изменения магнитного потока Φ сквозь поверхность, натянутую на контур с постоянным током, при его перемещении: ΔΦ = Φкон – Φнач.

Примеры решения задач

Пример8.1

В одной плоскости с длинным* прямым проводником, по которому идёт ток I1 = 10 А, находится плоская прямоугольная рамка. Длинные стороны рамки

параллельны проводу, расстояния от них до провода

a = 5 см,

b = 10 см. По рамке протекает ток I1 = 2 А, его направление

в

ближайшей длинной стороне совпадает с направлением

тока в

проводе. Рамка может растягиваться за счёт перемещения

её короткой

стороны MN параллельно самой себе. Найти работу сил

Ампера при

удлинении рамки на c = 2,0 см.

При перемещении стороны рамки MN она заметает

площадь S в

виде прямоугольника MNM’N’ (рис. 8.1). Работа сил Ампера

равна

A = I2зам|, где Φзам – магнитный поток через площадь S

зам = S BdS .

по

Направление B – магнитной индукции, созданной

проводом с

током – от нас в пределах площади S, B = μ0I/(2πr), где r

Рис. 8.1

расстояние

от прямого провода (см. пример 6.2).

Вектор dS параллелен B , а так как нас интересует |Φзам|, направим его коллинеарно B . Тогда

BdS = BdS . Модуль dS – это площадь бесконечно малого участка площади S прямоугольника, в пределах которого магнитную индукцию B можно считать постоянной. Эти участки представляют собой прямоугольники бесконечно малой ширины dr и высоты c и показаны на рис. 8.1, dS = cdr. Тогда

Φзам

a

2

2

ln

b

μ I cdr

μ I c

b

=

0 πr1

= 0πr1

a

,

где пределы интегрирования соответствуют положению на оси r длинный сторон прямоугольника. Работа

A= 2πr0

I1I2cln a =2,8 106

Дж.

μ

b

Сила Ампера F , действующая на рассматриваемую сторону, направлена в сторону перемещения рамки (рис. 8.1), следовательно, её работа положительна.

Задачи

8.1.Квадратная рамка со стороной a = 20 см помещена в однородное магнитное поле с магнитной индукцией B = 0,040 Тл. Нормаль к плоскости рамки составляет угол α = 60° с направлением вектора магнитной индукции. Найти магнитный поток, пронизывающий рамку.

8.2.Обмотка соленоида длиной l = 1,0 м и радиусом r = 25 мм содержит n = 11 витков/см, обтекаемых током I = 2,0 А. Считая, что витки плотно прилегают друг к другу, найти потокосцепление соленоида.

8.3. Обмотка тора (кольцевого соленоида) квадратного сечения со стороной a = 4 см состоит из N = 1000 витков, по которым идет ток I = 2,0 А. Внутренний радиус тора r1 = 8 см. Найти магнитный поток внутри тора.

65

8.4.По длинному* медному проводу кругового сечения идет ток I = 10 А. Плотность тока постоянна, относительная магнитная проницаемость меди μ = 1. Найти магнитный поток, приходящийся на единицу длины, в пределах самого провода.

8.5.Ток I = 5 А течёт по внутреннему медному проводу длинного* коаксиального кабеля и

возвращается по его внешней металлической оболочке. Диаметр внутреннего провода d1 = 2 мм, внешней оболочки – d2 = 10 мм. Найти магнитный поток, приходящийся на единицу длины кабеля. Плотность тока во внутреннем проводе постоянна по сечению.

8.6.Прямоугольная рамка со сторонами a = 10 см и b = 5 см лежит в одной плоскости с длинным* прямым проводом, по которому идет ток I = 10 А. Длинные стороны рамки параллельны проводу,

ближайшая сторона находится на расстоянии x1 = 5 см от прямого проводника. Рамка, оставаясь в той же плоскости, перемещается параллельно самой себе, удаляясь от прямого провода так, что

кратчайшее расстояние между ними возрастает до x2 = 10 см. Найти изменение магнитного потока, пронизывающего рамку, если ток, обтекающий рамку, по стороне, ближайшей к проводу, направлен: а) так же, как ток I; б) противоположно току I.

8.7.В однородном магнитном поле с индукцией B = 2,0·10–5 Тл в плоскости, перпендикулярной линиям индукции, проложены длинные* параллельные шины. Шины соединены подвижным проводником, длина которого l = 0,5 м равна расстоянию между ними. При токе I = 5 А проводник под действием силы Ампера перемещается на расстояние x = 4,0 см. Найти работу силы Ампера при этом перемещении. Ток в цепи считать постоянным.

8.8.Два параллельных достаточно длинных* провода находятся на расстоянии h1 = 2 см друг от друга. По проводам идут токи I1 = I2 = 2,0 А взаимно противоположного направления. Какую работу на единицу длины проводов совершают силы Ампера при медленном удалении проводов друг от друга до расстояния h2 = 8 см?

8.9.В плоскости, перпендикулярной линиям индукции однородного магнитного поля с индукцией B = 3,0·10–5 Тл лежит тонкое* кольцо, радиус которого r = 5 см. По кольцу идёт ток I = 1,5 А.

1. Каково должно быть направление тока в кольце, чтобы работа внешних сил при повороте кольца вокруг его диаметра на 180° была положительной?

2. Какую работу совершат внешние силы при этом повороте?

8.10.В однородном магнитном поле с индукцией B = 2,0·10–5 Тл помещена квадратная рамка со

стороной a = 10 см, обтекаемая током I = 1,0 А. Магнитный момент рамки параллелен B . Какую работу совершают силы Ампера при медленном повороте рамки вокруг одной из её сторон на угол

α = 90°; 180°; 360°?

8.11. В одной плоскости с длинным* прямым проводом, по которому идёт ток I1 = 10 А, находится плоская прямоугольная рамка со сторонами a = 10 см, b = 7,5 см. Длинные стороны рамки параллельны прямому проводу, расстояние от него до ближайшей длинной стороны рамки x0 = 2,5 см. По рамке идет ток I2 = 1,0 А. В стороне, ближайшей к прямому проводу, ток I2 направлен противоположно току I1. Рассчитать, какую работу совершат силы Ампера при медленном повороте рамки на угол α = 180°: а) вокруг оси, параллельной прямому проводу и проходящей через середину рамки; б) вокруг оси, совпадающей с длинной стороной, ближайшей к прямому проводу.

8.12. В одной плоскости с длинным* прямым проводом, по которому идёт ток I1 = 10 А, находится прямоугольная рамка со сторонами a = 5 см, b = 10 см. Длинные стороны рамки параллельны прямому проводу и ближайшая отстоит от него на расстояние x1 = 5 см. Рамка обтекается током I2 = 3 А, в прямом проводнике и ближайшей к нему стороне рамки токи направлены одинаково. Какую работу совершают внешние силы при поступательном прямолинейном перемещении рамки в её плоскости в направлении нормали к проводу? Конечное расстояние от прямого провода до ближайшей к нему длинной стороны рамки x2 = 10 см.

8.13. В длинный* соленоид вдоль его оси медленно втягивается маленькая* плоская рамка. Обмотка рамки состоит из N = 10 витков площадью S = 2,0 см2 каждый. Ток в обмотке рамки I1 = 1,0 А. Обмотка

66

соленоида содержит n = 100 витков/см, по которым идёт ток I2 = 5 А. Какую работу совершают силы Ампера при перемещении рамки из середины основания соленоида в середину его оси? Плоскость рамки нормальна к оси соленоида, токи I1 и I2 направлены одинаково.

8.14.Маленькая* квадратная рамка из мягкой проволоки со стороной a = 3,0 см, обтекаемая током

I1 = 0,10 А, медленно втягивается в середину длинного* соленоида. Обмотка соленоида состоит из n = 10 витков/см, ток в соленоиде I = 1,0 А. Перемещение рамки происходит вдоль оси соленоида, её плоскость всё время перпендикулярна оси соленоида. Токи в соленоиде и в рамке направлены одинаково. Найти суммарную работу, совершенную силами Ампера при перемещении рамки из середины основания до середины оси и при деформации рамки, если диаметр соленоида D >> a.

8.15.Два плоских круглых витка радиуса r = 10 см каждый расположены параллельно друг другу на расстоянии a = 10 см, причём прямая, соединяющая центры витков, перпендикулярна плоскостям обоих витков. По виткам протекают токи

взаимопротивоположных направлений, но одинаковые

по величине

I1 = I2 = 2,0 А. Концентрично первому витку расположен

маленький

виток площадью S = 0,20 см2. По витку течёт ток I = 0,10 А,

направленный так же, как ток I1 (рис. 8.2). Какую работу

должны

совершить внешние силы, чтобы переместить маленький

виток

(параллельно самомусебе) в середину второго витка?

8.16. Две тонкие катушки с током, векторы магнитных

Рис. 8.2

моментов

которых коллинеарны и по модулю равны соответственно pm1 = 8·10–2 А·м2 и pm2 = 0,12 А·м2, удалены друг от друга на расстояние x1 = 1,0 м. Расстояние между центрами катушек x1 >> l, где l – длина каждой катушки. Найти: а) работу, которую совершат внешние силы, чтобы увеличить расстояние

между

центрами

катушек

до

x2 = 1,1 м;

б) силу взаимодействия катушек на расстоянии x1.

Ответы

8.1.

= Ba

α =

8.2.

= μ0πn r lI =

8.3.Φ= 2μ0 NIaln 1+ a =6,5 106 Вб.

πr1

8.4.= μ40I =1,0 106 Вб м. l π

8.5.= μ40I 1 +2ln d2 =2,1 106 Вб м. l π d1

8.6.а) ΔΦ= 2μ0 Ialn x1 (x2 +b)= −5,8 108 Вб;

πx2 (x1 +b)

ΔΦ

μ

ln

1

(

2

+b

)

5,8 10 Вб

(

)

x

x

б)

= π0 Ia

x2

x1

+b

=

8 .

8.7.

A

= IBlx

=

.

8.8.

l

=

π0

I1I2 ln h12

=1,1 106 Дж м.

A

μ

h

n0

2

8.9. 1.

↑↑B .

2.

A′=

πBIr

=

.

A2 = − BIa = −

8.10.

A1 = −BIa

= −

;

; A3 = 0.

67

8.11.а) A = μ0 I1I2aln 1 + b0 =5,5 107 Дж;

πx

б) A =

π0

I1I2aln bx0

=1,4 107 Дж.

μ

b+ x

μ

2

2

ln

(x

(

+a)0 x

1,7

10

Дж

8.12.

A′=

μ2

1

b

x

1

2

)

7

.

π0

I I

1 x

+a2 =

8.13.

A =

0 I1I2NnS =6 10 5 Дж.

8

Дж

0 1

2

2

π

2

8,7

10

8.14.

A =

μ2I I na

=

.

8.15.

A = μ I I

S

1

+ a2

0 1

r

1

1

r2

2

2

μ

1

pm

2

3

x

3

8.16.

а) A = π0 pm

x

=3,2 1011 Дж.

=4,8 1010 Дж.

б) F = π0

mx14 m2

=5,8 109 Н.

2

1

μ

p p

68

Соседние файлы в папке 3 — практ_зан

  • #
  • #

Общее представление об электромагнитном поле

Длительное время представления об электрическом и магнитном поле не связывались между собой. Практические эксперименты подтверждали, что каждое из них имеет свои особенные свойства. Исследования Фарадея и Максвелла показали, что существует электромагнитное поле, которое может проявлять себя как электрическое или магнитное. Его можно описать с мощью вектора напряжённости. Если знать величину и направление данного вектора, то можно рассчитать силу воздействия.

Первым из учёных, кто обратил внимание на взаимное влияние магнитного поля и тока был известный учёный Х. К. Эрстед. Он исследовал влияние проводника с текущим по нему током на положение стрелки компаса. После этого учёные стали систематически изучать различные варианты взаимодействия.

Ампер появился на свет в 1775 году в Лионе. С детства он проявлял страсть к математике. Будучи подростком, изучал труды Эйлера и Лагранжа. Профессором математики Ампер стал в 1809 году, а в 1814 году был избран в академию наук. Хотя он преимущественно занимался математикой, его интересовала физика и некоторые другие науки.

Ампер был не первым человеком, который проявил интерес к связи магнитных и электрических полей, однако он впервые постарался найти точное математическое описание происходящих процессов. Им был не только установлен факт взаимодействия между электрическими токами, но и сформулирован закон данного явления.

Ампер доказал, что проводники начинают взаимодействовать, если по каждому из них протекает ток. В этом случае между ними возникают силы отталкивания или притягивания. В 1826 году Ампер впервые опубликовал результаты своего исследования, с помощью которого он изучал взаимодействие параллельных токов.

На рисунке ниже представлена схема одного из экспериментов Ампера, с помощью которого измеряется сила, действующая на проводник с током в магнитном поле. Стрелка красного цвета на рисунке показывает направление тока.

Основываясь на экспериментах, учёный сделал предположение, которое впоследствии получило название «гипотеза Ампера». Понимая, как влияет ток на магнитное поле, он сумел доказать, что вещество состоит из совокупности чрезвычайно маленьких круговых токов. Каждый из них порождает очень слабое магнитное поле. Поскольку все эти токи ориентированы хаотично, то внешне магнитное поле практически не проявляется. Однако в магнитах все круговые токи одинаково направлены и их воздействие складывается. Этим объясняются их особые свойства и практическое использование.

Ампер, используя свой закон, также объяснил эффект намагничивания. Согласно ему, у некоторых веществ под воздействием магнитного поля происходит упорядочивание круговых токов, и они постепенно ориентируются в одну сторону.

Эта гипотеза стала одним из источников теории магнетизма. Она смогла объяснить явление только частично, так как не дала ответа на вопрос о том, почему некоторые вещества подвергаются воздействию внешнего магнитного поля незначительно. Также остался необъяснённым вопрос, почему при намагничивании одни вещества создают магнитный поток сонаправленный внешнему полю (парамагнетики), а другие — противоположно направленный (диамагнетики).

Что такое сила Ампера

Собственно сила ампера и является той силой действия магнитного поля на проводник, по которому идет ток. Сила Ампера вычисляется по формуле как результат умножения плотности тока, идущего по проводнику на индукцию магнитного поля, в котором находится проводник. Как результат формула силы Ампера будет выглядеть так

са=ст*дчп*ми

Где, са – сила Ампера, ст – сила тока, дчп – длина части проводника, ми – магнитная индукция.

Значение закона Ампера

Сила Ампера – сила, действующая на проводник тока, находящийся в магнитном поле и равная произведению силы тока в проводнике, модуля вектора индукции магнитного поля, длины проводника и синуса угла между вектором магнитного поля и направлением тока в проводнике.

Для прямолинейного проводника сила Ампера имеет вид:

где: — сила тока, которая течет в проводнике, — вектор индукции магнитного поля, в которое проводник помещен, — длина проводника в поле, направление задано направлением тока, — угол между векторами .

Этой формулой можно пользоваться:

  • если длина проводника такая, что индукция во всех точках проводника может считаться одинаковой;
  • если магнитное поле однородное (тогда длина проводника может быть любой, но при этом проводник целиком должен находиться в поле).

Если размер проводника произволен, а поле неоднородно, то формула выглядит следующим образом:

На основании закона Ампера устанавливают единицы силы тока в системах СИ и СГСМ. Так как ампер равен силе постоянного тока, который при течении по двум параллельным бесконечно длинным прямолинейным проводникам бесконечно малого кругового сечения, находящихся на расстоянии 1м друг от друга в вакууме вызывает силу взаимодействия этих проводников равную на каждый метр длины.

Ток в один ампер – это такой ток, при котором два однородных параллельных проводника, расположенные в вакууме на расстоянии один метр друг от друга взаимодействуют с силой Ньютона.

Закон взаимодействия токов – два находящихся в вакууме параллельных проводника, диаметры которых много меньше расстояний между ними, взаимодействуют с силой прямо пропорциональной произведению токов в этих проводниках и обратно пропорциональной расстоянию между ними.

Формула силы Ампера

С учетом всего перечисленного можно получить формулу силы Ампера, которую еще называют законом Ампера:

Модуль силы Ампера равен произведению силы тока в проводнике, вектора магнитной индукции, длины проводника и синуса угла между направлениями векторов магнитной индукции и тока.

Магнитная сила Ампера

Единицы измерения силы Ампера

Основной единицей измерения силы Ампер (как и любой другой силы) в системе СИ является: [FA]=H

В СГС: [FA]=дин

Связь с другими единицами СИ

Если сила тока в проводнике равна 1 амперу, то за одну секунду через поперечное сечение проходит заряд, равный 1 кулону.

Разность потенциалов в 1 вольт на концах проводника с электрическим сопротивлением 1 ом создаёт в нём ток 1 ампер.

Если конденсатор ёмкостью в 1 фарад заряжать током 1 ампер, то напряжение на обкладках будет возрастать на 1 вольт каждую секунду.

Если изменять ток со скоростью 1 ампер в секунду в проводнике, имеющем индуктивность 1 генри, в нём создаётся ЭДС индукции, равная одному вольту.

Направление силы Ампера

Принимая к сведению то, что сила – векторная величина, определим её направление. Рассмотрим случай, когда проводник с током расположен между двумя полюсами магнитов под прямым углом к линиям магнитной индукции.

Выше мы установили, что согласно закону Ампера, действующая на данный проводник сила, равна: F = B*L*I. Направление вектора рассматриваемой силы определяется по результатам векторного произведения:

Амперова сила

Если полюса магнита статичны (неподвижны), то векторное произведение будет зависеть только от параметров электричества, в частности, от того, в какую сторону оно течёт.

Направление силы Ампера определяют по известному правилу левой руки: ладонь располагают навстречу магнитным линиям, а пальцы размещают вдоль проводника, в сторону устремления тока. На ориентацию силы Ампера указывает большой палец, образующий прямой угол с ладонью (см. рис. 4).

Интерпретация правила
Рис. 4. Интерпретация правила

Измените мысленно направление электрического тока, и вы увидите, что направление вектора Амперовой силы изменится на противоположное. Модуль вектора имеет прямо пропорциональную зависимость от всех сомножителей, но на практике эту величину удобно регулировать путём изменения параметров в электрической цепи (например, для регулировки мощности электродвигателя).

Правило левой руки

Формулировка правила левой руки для силы ампера звучит так:

Если расположить левую руку так, чтобы четыре пальца были направлены по направлению движения тока в проводнике, а перпендикулярная составляющая индукции $B_{perp}$ входила в ладонь, то отставленный большой палец покажет направление силы Ампера.

Как пользоваться этим правилом? Разберем примеры.

  • Допустим, проводник расположен горизонтально, и ток по нему идет вперед. Следовательно, четыре пальца левой руки надо вытянуть вперед по этому направлению.
  • Теперь допустим, что линии магнитного поля направлены сверху вниз (сверху «север» подковообразного магнита, снизу — «юг»). Следовательно, левую руку надо повернуть ладонью вверх, чтобы линии магнитного поля входили в ладонь и «прокалывали» ее (четыре пальца по-прежнему должны быть вытянуты вперед).
  • Отставленный большой палец левой руки будет направлен влево. Это и есть направление силы Ампера для данной ситуации.

Другой пример.

  • Пусть проводник расположен вертикально. А магнитное поле направлено справа налево (справа «север» магнита, слева — «юг»).
  • Располагаем левую руку четырьмя пальцами вверх. Ладонь открытой стороной должна «смотреть вправо», чтобы магнитные линии входили и «прокалывали» ее.
  • Отставленный большой палец покажет назад. Именно так и будет направлена сила Ампера в данном случае.

Обратите внимание, что силу Ампера порождает только перпендикулярная составляющая магнитного поля. А значит, руку надо располагать так, чтобы линии магнитного поля всегда входили в нее под углом, максимально близким к прямому.

Особым случаем является ситуация, когда направление тока и магнитной индукции совпадает. В этом случае руку невозможно расположить так, чтобы линии магнитной индукции входили в нее. Следовательно, силы Ампера здесь не возникнет. В самом деле, если линии магнитной индукции параллельны направлению тока, то перпендикулярная составляющая этих линий равна нулю, и значение силы Ампера в вышеприведенной формуле также равно нулю.

Различные случаи применения правила левой руки
Рис. 3. Различные случаи применения правила левой руки.Заключение

Работа силы Ампера

Проводники, на которые действует сила Ампера, могут перемещаться под действием этой силы. В этом случае говорят, что сила Ампера совершает работу. Из курса механики вспомним, что работа равна:

A=Fscos.α

F — сила, совершающая работу, s — перемещение, совершенное телом под действием этой силы, α — угол между вектором силы и вектором перемещения.

Отсюда работа, совершаемая силой Ампера, равна:

A=FAscos.α=BIlsin.βscos.α

α — угол между вектором силы и вектором перемещения, β — угол между условным направлением тока и вектором магнитной индукции.

Пример №3. Проводник длиной l = 0,15 м перпендикулярен вектору магнитной индукции однородного магнитного поля, модуль которого B = 0,4 Тл. Сила тока в проводнике I = 8 А. Найдите работу, которая была совершена при перемещении проводника на 0,025 м по направлению действия силы Ампера.

Так как проводник расположен перпендикулярно вектору магнитной индукции, и поле однородно, то синус угла между ними равен «1». Так как направление перемещение проводника совпадает с направлением действия силы Ампера, то косинус угла между ними тоже равен «1». Поэтому формула для вычисления работы силы Ампера принимает вид:

A=BIls

Подставим известные данные:

A=0,4·8·0,15·0,025=0,012 (Дж)=12 (мДж)

Задание EF17704

Магнитная сила Ампера
Как направлена сила Ампера, действующая на проводник № 3 со стороны двух других (см. рисунок), если все проводники тонкие, лежат в одной плоскости и параллельны друг другу? По проводникам идёт одинаковый ток силой I.

а) вверх

б) вниз

в) к нам

г) от нас

Алгоритм решения

1.Определить направление вектора результирующей магнитной индукции первого и второго проводников в любой точке третьего проводника.2.Используя правило левой руки, определить направление силы Ампера, действующей на третий проводник со стороны первых двух проводников.

Решение

На третьем проводнике выберем произвольную точку и определим, в какую сторону в ней направлен результирующий вектор →B, равный геометрической сумме векторов магнитной индукции первого и второго проводников (→B1и →B2). Применим правило буравчика. Мысленно сопоставим острие буравчика с направлением тока в первом проводнике. Тогда направление вращения его ручки покажем, что силовые линии вокруг проводника 1 направляются относительно плоскости рисунка против хода часовой стрелки. Ток во втором проводнике направлен противоположно току в первом. Следовательно, его силовые линии направлены относительно плоскости рисунка по часовой стрелке.

Магнитная сила Ампера

В точке А вектор →B1 направлен в сторону от наблюдателя, а вектор →B2— к наблюдателю. Так как второй проводник расположен ближе к третьему, создаваемое им магнитное поле в точке А более сильное (силы тока во всех проводниках равны по условию задачи). Следовательно, результирующий вектор →B направлен к наблюдателю.

Теперь применим правило левой руки. Расположим ее так, чтобы четыре пальца были направлены в сторону течения тока в третьем проводнике. Ладонь расположим так, чтобы результирующий вектор →B входил в ладонь. Теперь отставим большой палец на 90 градусов. Относительно рисунка он покажет «вверх». Следовательно, сила Ампера →FА, действующая на третий проводник, направлена вверх.

Ответ: а

Задание EF18417

Чему равна сила Ампера, действующая на стальной прямой проводник с током длиной 10 см и площадью поперечного сечения 2⋅10–2 мм2 , если напряжение на нём 2,4 В, а модуль вектора магнитной индукции 1 Тл? Вектор магнитной индукции перпендикулярен проводнику. Удельное сопротивление стали 0,12 Ом⋅мм2/м.

Алгоритм решения

1.Записать исходные данные и перевести единицы измерения величин в СИ.2.Записать формулу для определения силы Ампера.3.Выполнить решение в общем виде.4.Подставить известные данные и вычислить искомую величину.

Решение

Запишем исходные данные:

• Длина проводника: l = 10 см.• Площадь поперечного сечения проводника: S = 2⋅10–2 мм2.• Напряжение в проводнике: U = 2,4 В.• Модуль вектора магнитной индукции: B = 1 Тл.• Удельное сопротивление стали: r = 0,12 Ом⋅мм2/м.• Угол между проводником с током и вектором магнитной индукции: α = 90о.

10 см = 0,1 м

Сила Ампера определяется формулой:

FA=BIlsin.α

Так как α = 90о, синус равен 1. Тогда сила Ампера равна:

FA=BIl

Силу тока можно выразить из закона Ома:

I=UR..

Сопротивление проводника вычисляется по формуле:

R=rlS..

Тогда сила тока равна:

I=USrl..

Конечная формула для силы Ампера принимает вид:

FA=BlUSrl..=BUSr..=1·2,4·2·10−20,12..=0,4 (Н)

.

.

Ответ: 0,4
Задание EF17725

Магнитная сила Ампера
На непроводящей горизонтальной поверхности стола лежит жёсткая рамка массой m из однородной тонкой проволоки, согнутая в виде квадрата AСDЕ со стороной a(см. рисунок). Рамка находится в однородном горизонтальном магнитном поле, вектор индукции B которого перпендикулярен сторонам AE и CD и равен по модулю В. По рамке течёт ток в направлении, указанном стрелками (см. рисунок). При какой минимальной силе тока рамка начнет поворачиваться вокруг стороны CD?

Алгоритм решения

1.Сделать список известных данных.2.Определить, при каком условии рамка с током будет вращаться вокруг стороны CD.3.Выполнить решение в общем виде.

Решение

По условию задачи известными данными являются:

• Сторона квадратной рамки с током: a.• Вектор магнитной индукции однородного горизонтального магнитного поля, в котором лежит рамка: B.• Масса рамки: m.

Пусть по рамке течёт ток I. На стороны АЕ и CD будут действовать силы Ампера:

FA1=FA2=IaB

Для того чтобы рамка начала поворачиваться вокруг оси CD, вращательный момент сил, действующих на рамку и направленных вверх, должен быть не меньше суммарного момента сил, направленных вниз. Момент силы Ампера относительно оси, проходящей через сторону CD:

MA=Ia2B

Момент силы тяжести относительно оси CD:

Mmg=−12..mga

Чтобы рамка с током оторвалась от горизонтальной поверхности, нужно чтобы суммарный момент сил был больше нуля:

MA+Mmg>0

Так как момент силы тяжести относительно оси CD отрицательный, это неравенство можно записать в виде:

Ia2B>12..mga

Отсюда выразим силу тока:

I>mga2a2B..

I>mg2aB..

Практическое применение

Сила Ампера используется практически во всех электромеханических устройствах, где необходимо с помощью электрических процессов вызвать движение реальных объектов. Одним из примеров применения являются измерительные приборы.

При пропускании тока через проводник появляется магнитное взаимодействие, и рамка начинает вращаться. При этом смещается стрелка указателя до тех пор, пока сила Ампера не уравновесит силу упругости обеих пружин. Полученный показатель будет характеризовать ток, протекающий через проводник. Чем больше сила тока, тем сильнее отклонится стрелка.

На законе Ампера основывается и такая отрасль, как электротехника. Например, электромагнитная индукция применяется в электродвигателях.

Двигатель обеспечивает преобразование электроэнергии во вращение вала. Ток на рамку поступает через скользящие щётки. Она взаимодействует с постоянным магнитом, что приводит к её повороту под действием силы Ампера. В современных двигателях может использоваться одновременно несколько рамок. Это позволяет увеличить мощность мотора и сделать вращение оси более плавным.

Двигатели, работающие на основе рассматриваемого эффекта, активно используются в различных видах электротранспорта — трамваях, троллейбусах, электропоездах.

Еще одно применение закона Ампера — это громкоговорители. Внутри них находится постоянный магнит. Изменение силы тока вызывает изменение силы воздействия магнитного поля, что приводит к вибрации мембраны, производящей звук нужной частоты.

Притяжение между проводниками с током легло в основу точного определения единицы измерения 1 Ампер. При этом рассматривалась абстрактная ситуация, предполагающая наличие двух параллельно расположенных проводников с бесконечной длиной. Считалось, что каждый из них имеет бесконечно малое сечение, размерами которого можно пренебречь.

Принято, что на всём протяжении они находятся точно на расстоянии один метр и в вакууме. При прохождении тока величиной 1 Ампер сила, действующая на каждый метр проводников, должна составлять 0.0000002 Ньютона. Это определение вступило в силу в 1948 году.

Эксперимент

Для того чтобы иметь возможность своими глазами увидеть действие силы Ампера, можно провести дома небольшой эксперимент. Для начала необходимо взять магнит-подкову, в котором между полюсами поместить проводник. Всё желательно воспроизвести так, как на картинке.

Если замкнуть ключ, то можно увидеть, что проводник начнёт двигаться, смещаясь от начальной точки равновесия. Можно поэкспериментировать с направлениями пропускания тока и увидеть, что зависимо от направления движения меняется направление отклонения проводника. Из самого эксперимента можно вынести несколько наблюдений, которые подтверждают вышесказанное:

  • Магнитное поле действует исключительно на проводник с током.
  • На проводник с током в магнитном поле действует сила, которая является следствием их взаимодействия. Именно под воздействием этой силы проводник движется в пространстве в границах магнитного поля.
  • Характер взаимодействия прямо зависит от напряжения электрического тока и силовых линий магнитного поля.
  • Поле не действует на проводник с током, если ток в проводнике течёт параллельно направлению линий поля.

Предыдущая

ТеорияЗакон сохранения электрического заряда

Следующая

ТеорияПочему в странах разные розетки?

Формула силы Ампера в физике

Содержание:

  • Определение и формула силы Ампера
  • Закон Ампера
  • Силы, действующие на проводники с током в магнитном поле
  • Единицы измерения силы Ампера
  • Примеры решения задач

Определение и формула силы Ампера

Определение

Сила, действующая на проводник с током в магнитном поле, называется силой Ампера. Ее обозначения:
$bar{F}, bar{F}_A$ . Сила Ампера векторная величина. Ее направление определяет
правило левой руки: следует расположить ладонь левой руки так, чтобы силовые линии магнитного поля входили в нее.
Вытянутые четыре пальца указывали направление силы тока. В таком случае отогнутый на
большой палец укажет направление силы Ампера (рис.1).

Закон Ампера

Элементарная сила Ампера
($dbar{F}_A$) определена законом (или формулой) Ампера:

$$d bar{F}_{A}=I d bar{l} times bar{B}(1)$$

где I – сила тока,
$d bar{l}$ – малый элемент длины проводника – это вектор, равный
по модулю длине проводника, направленный в таком же направлении как вектор плотности тока,
$bar{B}$ – индукция магнитного поля, в которое помещен проводник с током.

Иначе эту формулу для силы Ампера записывают как:

$$d bar{F}_{A}=bar{j} times bar{B} d V(2)$$

где $bar{j}$ – вектор плотности тока, dV – элемент объема проводника.

Модуль силы Ампера находят в соответствии с выражением:

$$d F=I cdot B cdot d l cdot sin alpha(3)$$

где $alpha$ – угол между векторами магнитной индукции и направление течения тока. Из выражения (3) очевидно, что
сила Ампера максимальна в случае перпендикулярности линий магнитной индукции поля по отношению к проводнику с током.

Силы, действующие на проводники с током в магнитном поле

Из закона Ампера следует, что на проводник с током, равным I, действует сила равная:

$$bar{F}_{A}=I int_{l} d bar{l} times bar{B}(4)$$

где $bar{B}$ магнитная индукция, рассматриваемая в пределах малого кусочка проводника dl.
Интегрирование в формуле (4) проводят по всей длине проводника (l). {7}$ Гн/м(или Н/А2 ) – магнитная постоянная.
Проводники с токами одного направления притягиваются. Если направления токов в проводниках различны, то они отталкиваются.
Для рассмотренных выше параллельных проводников бесконечной длины сила Амперана единицу длины может быть вычислена по формуле:

$$frac{F}{l}=frac{mu_{0}}{2 pi} frac{I_{1} I_{2}}{d}$$

Формулу (6) в системе СИ применяют для получения количественного значения магнитной постоянной.

Единицы измерения силы Ампера

Основной единицей измерения силы Ампер (как и любой другой силы) в системе СИ является: [FA]=H

В СГС: [FA]=дин

Примеры решения задач

Пример

Задание. Прямой проводник длины l с током I находится в однородном магнитном поле B. На проводник
действует сила F. Каков угол между направлением течения тока и вектором магнитной индукции?

Решение. На проводник с током, находящийся в магнитном поле действует сила Ампера, модуль которой для
прямолинейного проводника с током расположенном в однородном поле можно представить как:

$$F=F_{A}=I B operatorname{lsin} alpha$$

где $alpha$ – искомый угол. Следовательно:

$$alpha=arcsin left(frac{F}{I B l}right)$$

Ответ. $alpha=arcsin left(frac{F}{I B l}right)$

236

проверенных автора готовы помочь в написании работы любой сложности

Мы помогли уже 4 396 ученикам и студентам сдать работы от решения задач до дипломных на отлично! Узнай стоимость своей работы за 15 минут!

Пример

Задание. Два тонких, длинных проводника с токами лежат в одной плоскости на расстоянии d друг от друга.
Ширина правого проводника равна a. По проводникам текут токи I

1 и I2 (рис.1). Какова, сила Ампера, действующая
на проводники в расчете на единицу длины?

Решение. За основу решения задачи примем формулу элементарной силы Ампера:

$$d bar{F}_{A}=I d bar{l} times bar{B}(2.1)$$

Будем считать, что проводник с током I1 создает магнитное поле, а другой проводник в нем находится. Станем искать силу
Ампера, действующую на проводник с током I2. Выделим в проводнике (2) маленький элемент dx (рис.1), который находится
на расстоянии x от первого проводника. Магнитное поле, которое создает проводник 1 (магнитное поле бесконечного прямолинейного проводника с
током) в точке нахождения элементаdxпо теореме о циркуляции можно найти как:

$$B cdot 2 pi x=mu_{0} I_{1} rightarrow B=frac{mu_{0} I_{1}}{2 pi x}$$

Вектор магнитной индукции в точке нахождения элемента dx направлен перпендикулярно плоскости
рисунка, следовательно, модуль элементарной силы Ампера, действующий на него можно представить как:

$$B cdot 2 pi x=mu_{0} I_{1} rightarrow B=frac{mu_{0} I_{1}}{2 pi x}$$

где ток, который течет в элементе проводника dx, выразим как:

$$B cdot 2 pi x=mu_{0} I_{1} rightarrow B=frac{mu_{0} I_{1}}{2 pi x}$$

Тогда выражение для dFA, учитывая (2. {a+b} frac{mu_{0} I_{1}}{2 pi x} cdot frac{I_{2}}{b} d x=frac{mu_{0} I_{1}}{2 pi} cdot frac{I_{2}}{b} ln left|frac{a+b}{a}right|$$

Проводники действуют друг на друга с силами равными по модулю и так как токи направлены одинаково, то они притягиваются.

Ответ. $F_{A}=frac{mu_{0} I_{1}}{2 pi} cdot frac{I_{2}}{b} ln left|frac{a+b}{a}right|$

Читать дальше: Формула силы выталкивания.

Сила Ампера | СПАДИЛО

Определение

Сила Ампера — сила, которая действует на проводник с током, помещенный в магнитное поле.

Модуль силы Ампера обозначается как FA. Единица измерения — Ньютон (Н).

Математически модуль силы Ампера определяется как произведение модуля вектора магнитной индукции B, силы тока I, длины проводника l и синуса угла α между условным направлением тока и вектором магнитной индукции:

FA=BIlsin.α

Максимальное значение сила Ампера принимает, когда ток в проводнике направлен перпендикулярно вектору магнитной индукции, так как sin.

90°=1. И сила Ампера отсутствует совсем, если ток в проводнике направлен относительно вектора магнитной индукции вдоль одной линии. В этом случае угол между ними равен 0, а sin.0°=1.

Пример №1. Максимальная сила, действующая в однородном магнитном поле на проводник с током длиной 10 см, равна 0,02 Н. Сила тока в проводнике равна 8 А. Найдите модуль вектора магнитной индукции этого поля.

10 см = 0,1 м

Так как речь идет о максимальной силе, действующей на проводник с током, тоsin.α при этом равен 1 (проводник с током расположен перпендикулярно вектору магнитной индукции).

Определение направления силы Ампера

Направление вектора силы Ампера определяется правилом левой руки.

Правило левой руки

Если левую руку расположить так, чтобы перпендикулярная проводнику составляющая вектора магнитной индукции →B входила в ладонь, то отогнутый на 90 градусов большой палец покажет направление силы, действующий на отрезок проводника (направление силы Ампера).

Пример №2. В однородном магнитном поле находится рамка, по которой начинает течь ток (см. рисунок). Какое направление (вверх, вниз, влево, вправо, от наблюдателя, наблюдателю) имеет сила, действующая на нижнюю сторону рамки?

Так как в нижней стороне рамки ток направлен вправо, то четыре пальца левой руки нужно направить вправо. Саму левую руку при этом нужно расположить перпендикулярно плоскости рисунка ладонью вверх, чтобы в нее входили линии вектора магнитной индукции. Если отогнуть большой палец на прямой угол, то он покажет направление силы Ампера, действующей на нижнюю часть рамки. В данном случае она направлена в сторону от наблюдателя.

Проводники, на которые действует сила Ампера, могут перемещаться под действием этой силы. В этом случае говорят, что сила Ампера совершает работу. Из курса механики вспомним, что работа равна:

A=Fscos.α

F — сила, совершающая работу, s — перемещение, совершенное телом под действием этой силы, α — угол между вектором силы и вектором перемещения.

Отсюда работа, совершаемая силой Ампера, равна:

A=FAscos.α=BIlsin.βscos.α

α — угол между вектором силы и вектором перемещения, β — угол между условным направлением тока и вектором магнитной индукции.

Пример №3. Проводник длиной l = 0,15 м перпендикулярен вектору магнитной индукции однородного магнитного поля, модуль которого B = 0,4 Тл. Сила тока в проводнике I = 8 А. Найдите работу, которая была совершена при перемещении проводника на 0,025 м по направлению действия силы Ампера.

Так как проводник расположен перпендикулярно вектору магнитной индукции, и поле однородно, то синус угла между ними равен «1». Так как направление перемещение проводника совпадает с направлением действия силы Ампера, то косинус угла между ними тоже равен «1». Поэтому формула для вычисления работы силы Ампера принимает вид:

A=BIls

Подставим известные данные:

A=0,4·8·0,15·0,025=0,012 (Дж)=12 (мДж)

Задание EF17704

Как направлена сила Ампера, действующая на проводник № 3 со стороны двух других (см. рисунок), если все проводники тонкие, лежат в одной плоскости и параллельны друг другу? По проводникам идёт одинаковый ток силой I.

а) вверх

б) вниз

в) к нам

г) от нас


Алгоритм решения

1.Определить направление вектора результирующей магнитной индукции первого и второго проводников в любой точке третьего проводника.

2.Используя правило левой руки, определить направление силы Ампера, действующей на третий проводник со стороны первых двух проводников.

Решение

На третьем проводнике выберем произвольную точку и определим, в какую сторону в ней направлен результирующий вектор →B, равный геометрической сумме векторов магнитной индукции первого и второго проводников (→B1и →B2). Применим правило буравчика. Мысленно сопоставим острие буравчика с направлением тока в первом проводнике. Тогда направление вращения его ручки покажем, что силовые линии вокруг проводника 1 направляются относительно плоскости рисунка против хода часовой стрелки. Ток во втором проводнике направлен противоположно току в первом. Следовательно, его силовые линии направлены относительно плоскости рисунка по часовой стрелке.

В точке А вектор →B1 направлен в сторону от наблюдателя, а вектор →B2— к наблюдателю. Так как второй проводник расположен ближе к третьему, создаваемое им магнитное поле в точке А более сильное (силы тока во всех проводниках равны по условию задачи). Следовательно, результирующий вектор →B направлен к наблюдателю.

Теперь применим правило левой руки. Расположим ее так, чтобы четыре пальца были направлены в сторону течения тока в третьем проводнике. Ладонь расположим так, чтобы результирующий вектор →B входил в ладонь. Теперь отставим большой палец на 90 градусов. Относительно рисунка он покажет «вверх». Следовательно, сила Ампера →FА, действующая на третий проводник, направлена вверх.

Ответ: а

pазбирался: Алиса Никитина | обсудить разбор | оценить

Задание EF18417

Чему равна сила Ампера, действующая на стальной прямой проводник с током длиной 10 см и площадью поперечного сечения 2⋅10–2 мм2 , если напряжение на нём 2,4 В, а модуль вектора магнитной индукции 1 Тл? Вектор магнитной индукции перпендикулярен проводнику. Удельное сопротивление стали 0,12 Ом⋅мм2/м.


Алгоритм решения

1.Записать исходные данные и перевести единицы измерения величин в СИ.

2.Записать формулу для определения силы Ампера.

3.Выполнить решение в общем виде.

4.Подставить известные данные и вычислить искомую величину.

Решение

Запишем исходные данные:

• Длина проводника: l = 10 см.

• Площадь поперечного сечения проводника: S = 2⋅10–2 мм2.

• Напряжение в проводнике: U = 2,4 В.

• Модуль вектора магнитной индукции: B = 1 Тл.

• Удельное сопротивление стали: r = 0,12 Ом⋅мм2/м.

• Угол между проводником с током и вектором магнитной индукции: α = 90о.

10 см = 0,1 м

Сила Ампера определяется формулой:

FA=BIlsin.α

Так как α = 90о, синус равен 1. Тогда сила Ампера равна:

FA=BIl

Силу тока можно выразить из закона Ома:

I=UR. .

Сопротивление проводника вычисляется по формуле:

R=rlS..

Тогда сила тока равна:

I=USrl..

Конечная формула для силы Ампера принимает вид:

FA=BlUSrl..=BUSr..=1·2,4·2·10−20,12..=0,4 (Н)

.

.

Ответ: 0,4

pазбирался: Алиса Никитина | обсудить разбор | оценить

Задание EF17725

На непроводящей горизонтальной поверхности стола лежит жёсткая рамка массой m из однородной тонкой проволоки, согнутая в виде квадрата AСDЕ со стороной a(см. рисунок). Рамка находится в однородном горизонтальном магнитном поле, вектор индукции B которого перпендикулярен сторонам AE и CD и равен по модулю В. По рамке течёт ток в направлении, указанном стрелками (см. рисунок). При какой минимальной силе тока рамка начнет поворачиваться вокруг стороны CD?


Алгоритм решения

1.Сделать список известных данных.

2.Определить, при каком условии рамка с током будет вращаться вокруг стороны CD.

3.Выполнить решение в общем виде.

Решение

По условию задачи известными данными являются:

• Сторона квадратной рамки с током: a.

• Вектор магнитной индукции однородного горизонтального магнитного поля, в котором лежит рамка: B.

• Масса рамки: m.

Пусть по рамке течёт ток I. На стороны АЕ и CD будут действовать силы Ампера:

FA1=FA2=IaB

Для того чтобы рамка начала поворачиваться вокруг оси CD, вращательный момент сил, действующих на рамку и направленных вверх, должен быть не меньше суммарного момента сил, направленных вниз. Момент силы Ампера относительно оси, проходящей через сторону CD:

MA=Ia2B

Момент силы тяжести относительно оси CD:

Mmg=−12..mga

Чтобы рамка с током оторвалась от горизонтальной поверхности, нужно чтобы суммарный момент сил был больше нуля:

MA+Mmg>0

Так как момент силы тяжести относительно оси CD отрицательный, это неравенство можно записать в виде:

Ia2B>12. .mga

Отсюда выразим силу тока:

I>mga2a2B..

I>mg2aB..

pазбирался: Алиса Никитина | обсудить разбор | оценить

Алиса Никитина | Просмотров: 9.3k

Формула закона Ампера — GeeksforGeeks

Согласно закону Ампера, магнитное поле, образованное электрическим током, пропорционально его величине. В этом случае константа пропорциональности равна проницаемости пустого пространства. Это также говорит о том, что магнитное поле связано с определенным током или наоборот, пока электрическое поле остается постоянным. Он используется для определения магнитной индукции, возникающей при использовании длинного провода с током. Он также используется для расчета магнитного поля, создаваемого длинным проводящим цилиндром с током.

Формула закона Ампера

Формула закона Ампера равна линейному интегралу магнитного поля вокруг замкнутого контура, равному количеству раз, которое алгебраическая сумма токов проходит через контур. Для проводника с током I, в котором поток тока создает магнитное поле вокруг провода, эту формулу можно использовать для расчета поля.

где,

μ или – постоянная магнитной проницаемости со значением 4π × 10 -7 N/A 2 ,

B – магнитное поле,

I – поток тока, проходящий через замкнутый контур,

L – длина петли.

Для замкнутого провода значение  равно 2πr. Итак, величина магнитного поля в этом случае равна

B = μ o I/2πr

0,2 м, если через него протекает ток силой 2 А.

Решение:

.

 = 2πr

= 2 (22/7) (0,2)

= 1,25 м

Используя формулу, которую мы имеем, ) (2)/(1.25)

= 2,011 × 10 -6 T

Задача 2. Найти магнитное поле замкнутого провода радиусом 0,5 м, если по нему течет ток силой 3 А .

Решение:

.

 = 2πr

= 2 (22/7) (0,5)

= 6,28 м

Используя формулу, которую мы имеем, ) (3)/(6.28)

= 6 × 10 -7 T

Задача 3. Найти магнитное поле замкнутого провода радиусом 0,8 м, если по нему течет ток силой 5 А .

Решение:

.

 = 2πr

= 2 (22/7) (0,8)

= 5,02 м

Используя формулу, которую мы имеем, ) (5)/(5.02)

= 1,25 × 10 -6 T

Задача 4. Найти магнитное поле замкнутого провода радиусом 0,4 м, если по нему течет ток силой 10 А .

Решение:

.

 = 2πr

= 2 (22/7) (0,4)

= 2,51 м

Используя формулу, которую мы имеем, ) (10)/(2.51)

= 5 × 10 -6 T

Задача 5. Найти ток, протекающий по замкнутому проводу радиусом 0,7 м, если его поле равно 3,4 × 10 -6 T.

Решение:

Мы имеем,

R = 0,7

B = 3,4 × 10 -6

μ O = 4π × 10002029

μ o = 4π × 100020 -7 9 9 μ o = 4π -7 9 o = 4π × 100020 -7 9 o = 4.

В нашем случае длина петли равна

 = 2πr

= 2 (22/7) (0,7)

= 4,4 м

Используя формулу, которую мы имеем, 2πr

3,4 × 10 -6 = (4π × 10 -7 ) (I)/(2,51)

I = 85,3/12,57

I = 6,78 А

Задача 6. Найти силу тока, протекающего по замкнутому проводу радиусом 0,32 м, если его поле равно 2,76 × 10 -7 Тл.

7 2 Решение:

8 Мы имеем,

R = 0,32

B = 2,76 × 10 -7

μo = 4π × 10 -7

В нашем случае длина петли равен,

= 2πr

= 2. (22/7) (0,32)

= 2,011 м

Используя формулу, которую мы имеем,

B = μ O I/2πr

2,76 × 10 -7 = (4π × 10 -7 ) (I)/(2,011)

I = 5,55/12,57

I = 0,44 A

Задача 7. Найти радиус замкнутого провода, если его поле 8,21 × 10 -5 Тл и сила тока 7 А.

Решение:

3 = 8,21 × 10

-5

μ о = 4π × 10 -7

I = 7

Используя формулу, которую мы имеем,

B = μ о I/2πr

8,21 × 10 -5 = (4π × 10 -7 ) (7)/r 9 ×

10 -2 )/8.21

r = 0,017 м

12.6: Закон Ампера — Physics LibreTexts

  1. Последнее обновление
  2. Сохранить как PDF
  • Идентификатор страницы
    4425
    • OpenStax
    • OpenStax
    Цели обучения

    К концу этого раздела вы сможете:

    • Объяснять, как закон Ампера связывает магнитное поле, создаваемое током, со значением тока
    • Рассчитайте магнитное поле длинного прямого провода, тонкого или толстого, по закону Ампера

    Фундаментальное свойство статического магнитного поля заключается в том, что, в отличие от электростатического поля, оно не является консервативным. Консервативное поле — это такое поле, которое совершает одинаковую работу над частицей, перемещающейся между двумя разными точками, независимо от выбранного пути. Магнитные поля таким свойством не обладают. Вместо этого существует связь между магнитным полем и его источником, электрическим током. Он выражается через линейный интеграл от (vec{B}) и известен как закон Ампера . Этот закон также может быть выведен непосредственно из закона Био-Савара. Теперь рассмотрим этот вывод для частного случая бесконечного прямого провода.

    На рисунке (PageIndex{1}) показана произвольная плоскость, перпендикулярная бесконечному прямому проводу, ток которого I направлен за пределы страницы. Линии магнитного поля представляют собой окружности, направленные против часовой стрелки и центрированные на проводе. Для начала рассмотрим (oint vec{B} cdot dvec{l}) по замкнутым путям M и N . Обратите внимание, что один путь ( M ) охватывает провод, а другой ( N ) нет. Поскольку силовые линии круглые, (vec{B} cdot dvec{l}) является произведением B и проекции dl на окружность, проходящую через (dvec{l }). Если радиус этого конкретного круга равен r , проекция равна (rdtheta) и

    [vec{B} cdot dvec{l} = Br , dtheta.]

    Рисунок (PageIndex{1}): Ток I длинного прямого провода направлен за пределы страницы. Интеграл (oint dtheta) равен (2pi) и 0 соответственно для путей M и N.

    С (vec{B}), заданным уравнением 12.4.1,

    [oint vec{B} cdot dvec{l} = oint left(frac{mu_0 I}{2pi r}right) , r , dtheta = frac{mu_0 I}{2pi} oint dtheta.]

    Для пути M , который циркулирует по проводу, (oint_M dtheta = 2pi) и

    [oint_M vec{B} cdot dvec{l} = mu_0 I.]

    Путь N , с другой стороны, циркулирует как через положительный (против часовой стрелки), так и через отрицательный (по часовой стрелке) (dtheta) (см. рисунок (PageIndex{1})), и поскольку он замкнут, (oint_N dтета = 0). Таким образом, для пути Н ,

    [oint_N vec{B} cdot dvec{l} = 0.]

    Распространение этого результата на общий случай есть закон Ампера.

    Закон Ампера

    По произвольному замкнутому пути,

    [oint vec{B} cdot dvec{l} = mu_0 I]

    где I — полный ток, проходящий через любой открытый поверхность S , периметр которой является путем интегрирования. Необходимо учитывать только токи внутри пути интегрирования.

    Чтобы определить, является ли конкретный ток I положительный или отрицательный, согните пальцы правой руки в направлении пути интегрирования, как показано на рисунке (PageIndex{1}). Если I проходит через S в том же направлении, что и ваш вытянутый большой палец, I положителен; если I проходит через S в направлении, противоположном вашему вытянутому большому пальцу, это отрицательно.

    Стратегия решения задач: закон Ампера

    Чтобы рассчитать магнитное поле, создаваемое током в проводах, выполните следующие действия:

    1. Определите симметрию тока в проводах. Если симметрии нет, используйте закон Био-Савара для определения магнитного поля.
    2. Определите направление магнитного поля, создаваемого проводом(ами), по правилу правой руки 2.
    3. Выберите петлю пути, в которой магнитное поле либо постоянно, либо равно нулю.
    4. Рассчитать ток внутри контура.
    5. Вычислите линейный интеграл (oint vec{B} cdot dvec{l}) вокруг замкнутого контура.
    6. Приравнять (oint vec{B} cdot dvec{l}) к (mu_0 I_{enc}) к (mu_0 I_{enc}) и найти (vec {В}).
    Использование закона Ампера для расчета магнитного поля, создаваемого проводом

    Использование закона Ампера для расчета магнитного поля, создаваемого постоянным током I в бесконечно длинном тонком прямом проводе, как показано на рисунке (PageIndex{ 2}).

    Рисунок (PageIndex{2}): Возможные компоненты магнитного поля B из-за текущего I , который направлен за пределы страницы. Радиальная составляющая равна нулю, потому что угол между магнитным полем и траекторией прямой.

    Стратегия

    Рассмотрим произвольную плоскость, перпендикулярную проводу, с током, направленным за пределы страницы. Возможные компоненты магнитного поля в этой плоскости (B_r) и (B_{theta}) показаны в произвольных точках на окружности радиусом r с центром на проводе. Поскольку поле цилиндрически симметрично, ни (B_r), ни (B_{theta}) не меняются в зависимости от положения на этой окружности. Также из симметрии радиальные линии, если они есть, должны быть направлены либо все внутрь, либо все наружу от провода. Это означает, однако, что должен существовать чистый магнитный поток через произвольный цилиндр, концентричный проводнику. Радиальная составляющая магнитного поля должна быть равна нулю, поскольку (vec{B}_r cdot dvec{l} = 0). Следовательно, мы можем применить закон Ампера к круговому пути, как показано на рисунке.

    Решение

    По этому пути (vec{B}) постоянна и параллельна (dvec{l}), поэтому

    [oint vec{B} cdot d vec{l} = B_{theta} oint dl = B_{theta}(2pi r).]

    Таким образом, закон Ампера сводится к

    [B_{theta}(2pi r ) = mu_0 I.]

    Наконец, поскольку (B_{theta}) является единственным компонентом (vec{B}), мы можем опустить индекс и написать

    [B = frac{mu_0 I}{2pi r}.]

    Это согласуется с приведенным выше расчетом Био-Савара.

    Значимость

    Закон Ампера хорошо работает, если у вас есть путь интегрирования, по которому (vec{B} cdot dvec{l}) дает результаты, которые легко упростить. Для бесконечного провода это легко работает с круговым путем вокруг провода, так что магнитное поле не учитывается при интегрировании. Если зависимость от пути кажется сложной, вы всегда можете вернуться к закону Био-Савара и использовать его для нахождения магнитного поля.

    Пример (PageIndex{2}): Расчет магнитного поля толстой проволоки по закону Ампера

    Радиус длинного прямого провода на рисунке (PageIndex{3}) равен a , и по проводу течет ток (I_0), равномерно распределенный по его поперечному сечению. Найдите магнитное поле как внутри, так и снаружи провода.

    Рисунок (PageIndex{3}): (a) Модель провода с током радиусом a и током (I_0). (b) Поперечное сечение того же провода с радиусом х и петлей Ампера радиусом х .

    Стратегия

    Эта задача имеет ту же геометрию, что и пример (PageIndex{1}), но замкнутый ток изменяется по мере того, как мы перемещаем путь интегрирования снаружи провода внутрь провода, где он не захватывает весь ток прилагается (см. рисунок (PageIndex{3})).

    Решение

    Для любого кругового пути радиусом r с центром на проводе

    [oint vec{B} cdot dvec{l} = oint Bdl = Boint dl = B(2pi r). ]

    Согласно закону Ампера это равно полному току, проходящему через любую поверхность, ограниченную путем интегрирования. 92} (r leq a).]

    Вне провода ситуация идентична ситуации бесконечного тонкого провода из предыдущего примера; то есть

    [B = frac{mu_0 I_0}{2pi r} (r geq a).]

    Вариант B с r показан на рисунке ( Индекс страницы{4}).

    Рисунок (PageIndex{4}): Изменение магнитного поля, создаваемого током (I_0) в длинном прямом проводе радиусом a .

    Значение

    Результаты показывают, что по мере увеличения радиального расстояния внутри толстой проволоки магнитное поле увеличивается от нуля до известного значения магнитного поля тонкой проволоки. Вне провода поле падает независимо от того, толстый провод или тонкий.

    Этот результат аналогичен тому, как закон Гаусса для электрических зарядов ведет себя внутри равномерного распределения зарядов, за исключением того, что закон Гаусса для электрических зарядов имеет равномерное объемное распределение заряда, тогда как закон Ампера здесь имеет однородную область распределения тока. Кроме того, спад вне толстого провода подобен тому, как спадает электрическое поле вне линейного распределения заряда, поскольку оба случая имеют одинаковую геометрию, и ни один случай не зависит от конфигурации зарядов или токов, когда петля находится снаружи. распространение.

    Использование закона Ампера с произвольными путями

    Используйте закон Ампера для оценки (oint vec{B} cdot dvec{l}) для текущих конфигураций и путей на рисунке (PageIndex{5} ).

    Рисунок (PageIndex{5}): Текущие конфигурации и пути для примера (PageIndex{3}).

    Стратегия

    Закон Ампера гласит, что (oint vec{B} cdot dvec{l} = mu_0 I), где I — это полный ток, проходящий через замкнутый контур. Самый быстрый способ вычислить интеграл — вычислить (mu_0 I), найдя чистый ток через контур. Положительные токи текут по большому пальцу правой руки, если ваши пальцы скручиваются в направлении петли. Это подскажет нам знак ответа.

    Решение

    (a) Ток, протекающий вниз по контуру, равен току, выходящему из контура, поэтому чистый ток равен нулю. {-6} T cdot m.) 9{-6} T cdot m).

    Значение

    Если бы все токи закручивались таким образом, что один и тот же ток входил бы в петлю и из петли, чистый ток был бы равен нулю и магнитное поле не присутствовало бы. Вот почему провода в электрическом шнуре расположены очень близко друг к другу. Токи, протекающие к устройству и от устройства по проводу, равны нулевому общему току, протекающему через петлю Ампера вокруг этих проводов. Следовательно, никакие блуждающие магнитные поля не могут присутствовать в проводах с током.

    Упражнение (PageIndex{1})

    Попробуйте использовать закон Ампера для расчета магнитных полей конечного прямого провода и круглой проволочной петли. Почему это не полезно для этих расчетов?

    Ответить

    В этих случаях интегралы вокруг петли Ампера очень сложны из-за отсутствия симметрии, поэтому этот метод бесполезен.

    Сэмюэля Дж. Линга (Государственный университет Трумэна), Джеффа Санни (Университет Лойолы Мэримаунт) и Билла Мёбса с многими соавторами.

    Содержание:

    Сила и закон Ампера:

    Действие магнитного поля на проводник с током в 1820 г. исследовал экспериментально Андре Мари Ампер. Меняя форму проводников и их расположение в магнитном поле, Ампер сумел определить силу, действующую на отдельный участок проводника с током (элемент тока). В его честь ее назвали силой Ампера.

    Исследуем с помощью динамометра модуль силы Ампера, действующей на участок прямолинейного проводника длиной I с током силой l со стороны магнитного поля индукцией В (рис. 150).

    Сила и закон Ампера - формулы и определение с примерами

    Согласно экспериментальным данным и результатам вычислений модуль силы:

    • пропорционален длине проводника, находящегося в магнитном поле (F ~ l)
    • пропорционален модулю индукции магнитного поля (F ~ В); пропорционален силе тока в проводнике (F ~ l);
    • зависит от ориентации проводника в магнитном поле, т. е. от угла Сила и закон Ампера - формулы и определение с примерами

    Обобщая полученные результаты, запишем выражение для силы Ампера Сила и закон Ампера - формулы и определение с примерами
    в виде
    Сила и закон Ампера - формулы и определение с примерами
    где В — индукция магнитного поля, l — длина участка проводника, находящегося в магнитном поле, I — сила тока в проводнике, Сила и закон Ампера - формулы и определение с примерами — угол, образованный направлением тока и Сила и закон Ампера - формулы и определение с примерами

    Закон Ампера

    Это выражение называют законом Ампера:

    • модуль силы, с которой магнитное поле действует на находящийся в нем прямолинейный проводник с током, равен произведению индукции В этого поля, силы тока I, длины участка проводника l и синуса угла между направлениями тока и индукции магнитного поля.

    Сила Ампера Сила и закон Ампера - формулы и определение с примерами всегда перпендикулярна направлению тока в проводнике и вектору индукции Сила и закон Ампера - формулы и определение с примерами магнитного поля. Для определения направления силы

    Правило левой руки

    Ампера используют правило левой руки (рис. 151):

    Сила и закон Ампера - формулы и определение с примерами
     

    если ладонь левой руки расположить так, чтобы перпендикулярная к проводнику составляющая вектора индукции магнитного поля входила в ладонь, а четыре вытянутых пальца указывали направление тока, то отогнутый на 90° большой палец укажет направление силы Ампера.

    Магнитное взаимодействие проводников с током используется для определения в СИ одной из основных единиц — единицы силы тока — ампера.

    Один ампер есть сила постоянного тока, поддерживаемого в каждом из двух прямолинейных параллельных проводниках бесконечной длины и ничтожно малого кругового сечения, расположенных на расстоянии 1 м один от другого в вакууме, который вызывает между этими проводниками силу взаимодействия, равную Сила и закон Ампера - формулы и определение с примерамиН на каждый метр длины проводников.

    Магнитное поле

    Обобщение учеными результатов теоретических и экспериментальных исследований различных взаимодействий в природе привело к выводу, что материя может существовать не только в форме вещества, по и в форме поля. Изучая физику в предыдущих классах, вы узнали о существовании электрического и магнитного полей, благодаря которым взаимодействуют наэлектризованные тела. Работы Дж. Максвелла, М. Фарадея и других ученых показали, что эти поля взаимосвязаны и фактически являются проявлениями более универсального электромагнитного поля. И только выбор системы отсчета определяет, что мы наблюдаем — электрическое или магнитное поле. Изучить все свойства электромагнитного поля довольно сложно. Поэтому в физике изучают постепенно отдельные проявления этого ноля. Одним из этапов изучения электромагнитного поля является изучение магнитного поля, которое проявляется в случае, когда заряженные частицы или тела в определенной системе отсчета движутся равномерно. В этом разделе рассматриваются не только условия, при которых магнитное поле наблюдается, но и физические величины, которые описывают его свойства, законы, по которым взаимодействуют магнитные поля и вещественные объекты. Знание этих законов позволяет производить важные для практики расчеты результатов взаимодействия магнитного поля с различными физическими телами.

    Явления, которые мы называем магнитными, известны человечеству очень давно. Необычные свойства магнетита (разновидности железной руды) использовались в Древнем Китае, а потом и в других странах для изготовления компасов. Магнитам приписывали магические свойства, их действием объясняли непонятные явления природы, пробовали лечить болезни.
    Систематизированные исследования магнитов провел английский физик У. Гильберт в XVI в. Он не только исследовал взаимодействие постоянных .магнитов, но и установил, что Земля является большим магнитом.

    Учение о магнитах развивалось длительное время обособленно, как отдельная отрасль науки, пока ряд открытий и теоретических исследований в XIX в. не доказали его органическую связь с электричеством.

    Одним из фундаментальных доказательств единства электрических и магнитных явлений является опыт Г.Х. Эрстеда, датского физика, который в 1820 г. заметил, что магнитная стрелка изменяет ориентацию вблизи проводника с током (рис. 2.1).

    Сила и закон Ампера - формулы и определение с примерами
    Pиc. 2.1. Опыт Эрстеде

    Было вполне очевидно, что причиной изменения ориентации стрелки является электрический ток -направленное движение заряженных частиц в проводнике. C подробным описанием этого опыта вы встречались в 9-м классе.

    Магнитное действие движущихся заряженных тел исследовал также американский физик Г. Роуланд в 1878 г. Основная часть его установки представляла собой эбонитовый диск 1, покрытый тонким слоем золота (рис. 2.2). Диск был насажен на вал и мог свободно вместе с ним вращаться между двумя стеклянными пластинами 2. Над эбонитовым диском были укреплены на тонкой нити две намагниченные стальные иголки 3, чувствительные к магнитному полю. Когда диску сообщили некоторый заряд и начали вращать, иголки повернулись на некоторый угол, что свидетельствовало о наличии магнитного поля. При увеличении скорости вращения иголки поворачивались на больший угол.

    Сила и закон Ампера - формулы и определение с примерами
    Рис. 22. Главная часть установки Роуланда по выявлению магнитного поля движущегося электрически заряженного диска

    Опытами Г. Роуланда было подтверждено открытие Эрстеда о связи магнитного поля с движущимися электрически заряженными частицами или телами.

    Сила и закон Ампера - формулы и определение с примерами Генри Роуланд (184β-1901) — американский физик; научные работы в области
    электродинамики, оптики, спектроскопии и теплоты. Он доказал, что заряженные
    тела, если они движутся, вызывают магнитное взаимодействие.

    Магнитные явления хотя и связаны с электрическими, но не идентичны им. Это подтверждают опыты.

    Если взять два длинных параллельных проводника и присоединить к источнику тока, то заметим, что они взаимодействуют между собой (рис. 2.3) в зависимости от направления тока в них. При токах противоположных направлений проводники отталкиваются (рис. 2.3-а). Если токи одного направления, то проводники притягиваются друг к другу (рис. 2.3-б).

    Сила и закон Ампера - формулы и определение с примерами
    Pиc. 23. Магнитное взаимодействие проводников с током

    Действие проводника с током на магнитную стрелку или другой проводник с током происходит при отсутствии непосредственного контакта между ними, благодаря наличию вокруг проводника магнитного поля.

    Магнитное поле имеет свои особенности, которые выделяют его среди других полей:

    1. магнитное поле наблюдается всегда, когда есть движущиеся заряженные частицы или тела;
    2. магнитное поле действует только на движущиеся заряженные тела или частицы.

    Другие свойства будут описаны далее.

    Магнитная индукция

    Наблюдения за магнитными взаимодействиями в лаборатории или в природе показывают, что действия магнитного поля па физические тела или проводники с током при равных условиях могут быть различными.

    Интенсивность магнитного взаимодействия может быть различной.

    Если для выявления магнитного поля Земли магнитную стрелку компаса приходится устанавливать на специальных опорах, которые существенно уменьшают силы трения, то действие электромагнита, в обмотках которого проходит электрический ток, будет заметным даже тогда, когда стрелка будет просто лежать на столе.

    Различным будет и взаимодействие параллельных проводников с током. Сила взаимодействия этих проводников будет изменяться, если будет изменяться сила тока в них или расстояние между ними, — она будет увеличиваться при увеличении силы тока или при уменьшении расстояния.

    Для всех таких случаев говорят о «сильном» или «слабом» поле. Аналогичные случаи рассматривались при изучении свойств электрического поля, при рассмотрении действия электрического поля на заряженные тела. Для количественной характеристики электрического поля введена напряженность электрического поля. Для магнитного же поля используется также силовая характеристика и соответствующая ей физическая величина магнитная индукция. Магнитная индукция является векторной величиной и обозначается буквой В. Поскольку для исследования магнитного поля длительное время пользовались магнитной стрелкой на острие, то магнитная индукция как характеристика магнитного поля была связана с действием магнитного поля па магнитную стрелку. Так, направление полюсов стрелки послужило базой для установления направления вектора магнитной индукции изучаемого поля. Условились, что за направление магнитной индукции принимается направление северного полюса стрелки.

    Магнитная индукция — векторная величина, имеющая направление.

    Исследуем с помощью магнитной стрелки магнитное поле проволочного витка с током.

    Замкнув цепь, в которую включен виток, начнем обносить магнитную стрелку на острие вокруг витка. Заметим, что ориентация стрелки при этом будет меняться. В разных точках она будет иметь различную ориентацию. Наиболее ощутимым будет действие поля на стрелку в центре витка (рис. 2.4).

    Сила и закон Ампера - формулы и определение с примерами
    Puc. 2.4. Продольная ось магнитной стрелки, находящаяся в центре витка с током, перпендикулярна его плоскости

    Таким образом, мы установили, что магнитная индукция витка или прямоугольной рамки будет иметь максимальное значение в центре.

    Продольная ось магнитной стрелки плоскости витка. Аналогичное явление будет наблюдаться и тогда, когда возьмем прямоугольную рамку или моток провода произвольной формы.

    В отличие от напряженности электрического поля магнитная индукция как векторная величина не совпадает по направлению с направлением силы, которая действует на проводник с током. Выясним, как направление вектора магнитной индукции зависит от направления тока в витке.

    Магнитная индукция — это силовая характеристика поля. Она определяет силу, которая действует на проводник с током или на движущуюся частицу.

    Отметив направление магнитной стрелки при определенном направлении тока в витке, изменим направление последнего на противоположное. Магнитная стрелка развернется на 1800, показывая, что направление магнитной индукции также изменилось. Таким образом, направление магнитной индукции витка с током зависит от направления тока и нем.

    Чтобы каждый раз, когда нужно знать направление магнитной индукции, не проводить опыты со стрелкой, пользуются правилом правого винта (буравчика).

    Это правило позволяет запомнить связь направления тока в витке с направлением магнитной индукции его поля. Для этого необходимо представить, как будет двигаться правый винт, приставленный перпендикулярно к плоскости витка, при вращении его по направлению тока в витке.

    Если направление вращения правого винта, расположенного в центре витка с током, совпадает с направлением тока, то его поступательное движение показывает направление магнитной индукции (рис. 2.5).

    Магнитное поле существует и вокруг прямого проводника с током. Для подтверждения этого магнитную стрелку будем обносить вокруг проводника, не изменяя расстояния (рис. 2.6).

    Сила и закон Ампера - формулы и определение с примерами

    Pиc. 2.5. Определение
    направления магнитной
    индукции витка с током

    Сила и закон Ампера - формулы и определение с примерами
    Pиc. 2.6. Исследование магнитного
    поля прямого проводника с током
    при помощи магнитной стрелки

    В разных точках ее ориентация будет различной, но ось стрелки всегда будет направлена по касательной к траектории движения.

    Соответственно и магнитная индукция проводника с током будет иметь такое ясе направление.
    При изменении направления тока в проводнике на противоположное стрелка развернется на 180° и покажет направление магнитной индукции, которое также будет противоположным к прежнему.

    Таким образом, направление магнитной индукции прямого проводника зависит от направления тока в нем. Для облегчения его определения, как и в предыдущем случае, на основании анализа результатов эксперимента, сформулировано правило правого винта (рис. 2.7): если направление поступательного движения правого винта совпадает с направлением тока в проводнике, то направление его вращения показывает направление магнитной индукции.

    Сила и закон Ампера - формулы и определение с примерами
    Pиc. 2.7. Определение направления магнитной индукции поля прямого проводника с током при помощи правою винта (буравчика)

    Для измерения магнитной индукции применяется специальная единица тесла (Тл). Эта единица названа в честь сербского ученого и изобретателя Николы Теслы.

    Сила и закон Ампера - формулы и определение с примерами Никола Тесла (1856-1943) — родился в Сербии, изобретатель и физик.
    Известен благодаря своим изобретениям в области электротехники
    и электроники; работал инженером на предприятиях Венгрии, Франции, США.

    В практике используются долевые величины:

    • 1 миллитесла = 1 мТл = 10-3 Тл,
    • 1 микротесла 1 мкТл 10-6 Тл.

    Значения магнитной индукции измеряют специальными приборами, которые называются магнитометрами или индикаторами магнитной индукции (рис. 2.8).

    Сила и закон Ампера - формулы и определение с примерами
    Pиc. 2.8. Лабораторный магнитометр для школьных опытов

    Часто вместо прямых измерений пользуются формулами, которые позволяют рассчитать магнитную индукцию на основании параметров проводника. Таким примером может быть расчет модуля магнитной индукции прямого проводника с током. Экспериментально подтверждено, что магнитная индукция поля прямого проводника с током прямо пропорциональна силе тока в проводнике и обратно пропорциональна расстоянию от его оси:

    Сила и закон Ампера - формулы и определение с примерами

    Магнитная индукция прямого проводника с током пропорциональна силе тока в нем и обратно пропорциональна расстоянию от проводника до точки наблюдения.

    Коэффициент пропорциональности в этой формуле зависит от выбора системы единиц измерений. В Международной системе единиц (СИ) он имеет значение Сила и закон Ампера - формулы и определение с примерами

    где μ0 — магнитная постоянная, ее числовое значение 1,256 × × 10-6 Н/А2.

    Сила и закон Ампера - формулы и определение с примерами

    Тогда окончательно для рассчетов модуля магнитной индукции поля прямого проводника с током имеем формулу:

    Сила и закон Ампера - формулы и определение с примерами

    где μ0 — магнитная постоянная; I — сила тока в проводнике: r — расстояние от проводника до данной точки поля.

    Пример №1

    Каково значение модуля магнитной индукции в точке поля, удаленной на 3 см от бесконечно длинного проводника, по которому проходит ток 6 А?

    Дано:
    r = 3 см,
    I = 6 А.
    Решение
    Магнитная индукция прямого проводника
    с током рассчитывается по формуле:
    Сила и закон Ампера - формулы и определение с примерами
    В — ?

    Подставив значения физических величин, получим
    Сила и закон Ампера - формулы и определение с примерами
    Сила и закон Ампера - формулы и определение с примерами

    Ответ: магнитная индукция поля прямого проводника с током равна 4 • 10-5 Тл.

    Действие магнитного поля на проводник с током и сила Ампера

    Поскольку вокруг проводников с током возникает магнитное поле, естественно предположить, что в магнитном поле на них действует сила.

    На проводник с током в магнитном поле действует сила.

    Проведем исследование с целью определения, от чего зависит модуль и направление этой силы. Для этого используем установку, в которой прямой проводник подвешен в магнитном поле постоянного магнита так, что его можно включать в электрическую цепь, силу тока в которой можно изменять при помощи реостата. Амперметр будет измерять силу тока в цепи.

    Замкнув электрическую цепь, заметим, что проводник отклонится от положения равновесия, а динамометр покажет некоторое значение силы. Увеличим силу тока в проводнике в 2 раза и увидим, что сила, действующая на проводник, также увеличится в 2 раза. Любые другие изменения силы тока будут вызывать соответствующие изменения силы. Сопоставление результатов всех измерений позволяет сделать вывод, что сила F, которая действует на проводник с током, пропорциональна силе тока к нем:
    F~I.

    Сила, действующая на проводник с током в магнитном поле, называется силой Ампера.

    Сила Ампера пропорциональна силе тока в проводнике.

    Сила и закон Ампера - формулы и определение с примерами
    Pиc. 2.9. Установка для изучения действия магнитного поля на проводник с током

    Расположим еще один магнит рядом с первым. Длина проводника, находящегося в магнитном поле, увеличится приблизительно в 2 раза. Значение силы, действующей на проводник, в этом случае также увеличится в 2 раза. Таким образом, сила FΔ, действующая на проводник с током в магнитном поле, пропорциональна длине проводника Δl, который расположен в магнитном поле:

    F~ΔI.

    Сила Ампера пропорциональна длине активной части проводника. 

    Сила увеличится также тогда, когда применим другой, более мощный магнит с большей магнитной индукцией поля.

    Это позволит сделать вывод, что сила Ампера FА зависит от магнитной индукции поля:

    F~B.

    Опыт позволяет убедиться и в том, что наибольшее значение силы Ампера будет тогда, когда угол между проводником и вектором магнитной индукции будет равен 90°. Если этот угол будет равен нулю, т. е. вектор магнитной индукции будет параллельным проводнику, то сила Ампера также будет равна нулю. Отсюда легко сделать вывод, что сила Ампера зависит от угла между вектором магнитной индукции и проводником.

    Окончательно для расчетов имеем формулу Сила и закон Ампера - формулы и определение с примерами

    Направление силы Ампера определяется по правилу левой руки (рис. 2.10): если левую руку разместить так, чтобы линии магнитной индукции входили в ладонь, а четыре от. ставленных пальца показывали направление тока в проводнике, то отставленный под углом 90″ большой палец покажет направление силы, действующей на проводник с током в магнитном поле.

    Сила и закон Ампера - формулы и определение с примерами
    Pиc. 2.10. При помощи левой pуκu можно определить направление силы Ампера

    Если левую руку разместить так. чтобы линии магнитной индукции входили в ладонь, а четыре отставленных пальца показывали направление тока в проводнике, то отставленный под углом 90° большой палец покажет направление силы, действующей на проводник с током в магнитном поле.

    Взаимодействие проводников с током

    Взаимодействие проводников с током объясняется действием силы Ампера (рис. 2.11).

    Каждый из проводников имеет свое магнитное поле, которое действует на соседний проводник с током и способствует появлению силы Ампера. Так, проводник AA‘ по которому проходит ток I1, имеет магнитное поле, модуль индукции B1 которого, как указывалось ранее, равен

    Сила и закон Ампера - формулы и определение с примерами

    где r — расстояние от проводника до точки наблюдения.

    Если проводник CC’ длиной Δl находитсяy на расстоянии r от проводника AA’ и в нем проходит ток I2, то на него действует сила Ампера FА, поскольку он находится в магнитном поле проводника AA’ . Значение этой силы равно Сила и закон Ампера - формулы и определение с примерами

    Поскольку проводники параллельны и угол между проводником CC’ и вектором магнитной индукции B равен 90°, то sinα = 1.

    Подставим в последнюю формулу значение магнитной индукции поля проводника AA’:

    Сила и закон Ампера - формулы и определение с примерами

    Силу взаимодействия двух параллельных проводников с током можно определить, зная только расстояние между ними и силу тока в них.

    Как и при любом взаимодействии, такая сила, согласно третьему закону Ньютона, действует на каждый из проводников. Только направления их противоположны.

    Таким образом, два параллельных проводника нзнимодей-ствуют между собой благодаря магнитным полям, которые образуются вокруг проводников, по которым проходит электрический ток.

    Пример №2

    Определить модуль силы Ампера, которая действует на проводник с током длиной 25 см в магнитном поле с индукцией 0,04 Тл, если между вектором магнитной индукции и направлением тока угол 30° сила тока в проводнике 0,25 А.

    Дано:
    ∆l = 25 см.
    В = 0,04 Тл,

    Сила и закон Ампера - формулы и определение с примерами = 30%
    I = 0,25 А.

    Решение
    На проводник с током в магнитном поле действует сила
    Сила и закон Ампера - формулы и определение с примерами

    Подставим значения всех величин:
    Сила и закон Ампера - формулы и определение с примерами

    FA— ?

    Ответ: модуль силы равен 1,25 • 10-3 Н.

    Использование действия силы Ампера

    Силу Ампера применяют для преобразования энергии электрического тока в механическую энергию проводника. Такое превращение происходит во многих электротехнических устройствах. Рассмотрим некоторые из них.

    Eлектроиэмеритальные приборы магнитоэлектрической системы

    Электроизмерительный прибор магнитоэлектрической системы состоит из постоянного магнита и проволочной рамки, расположенной между его полюсами (рис. 2.12). Полюса магнита имеют специальные насадки, создающие однородное магнитное поле, в котором вращение рамки не приводит к изменению угла между магнитной индукцией и проводниками рамки. Этот угол всегда равен 90°.

    Сила и закон Ампера - формулы и определение с примерами
    Pиc. 2.12. Устройство электроизмерительного прибора магнитоэлектрической системы

    C рамкой соединены две спиральные пружины, которые подводят к рамке электрический ток. Во время прохождения электрического тока по витком рамки возникает сила Ампера, пропорциональная силе тока в рамке. Чем больше сила действует на витки рамки, тем больше закручиваются спиральные пружины, которых возникает сила упругости. Когда сила Ампера и сила упругости станут равными, вращение рамки прекратится.

    Стрелка, прикрепленная к рамке, показывает угол поворота рамки. Этот угол пропорционален силе тока в рамке.

    Электрический двигатель постоянного тока

    Электрический двигатель применяют для преобразования энергии электрического тока в механическую энергию вращения вала двигателя. Принцип его действия подобен принципу действия электроизмерительного прибора магнитоэлектрической системы, описанного выше. Только в его конструкции отсутствует пружина, поэтому рамка может поворачиваться на любой угол. Электрический ток к рамке, размещенной на валу и имеющей стальной сердечник, подается через специальные скользящие контакты-щетки (рис. 2.13).

    Сила и закон Ампера - формулы и определение с примерами
    Рис. 213. Устройство двигателя постоянного тока

    При замыкании цепи питания двигателя ток проходит по рамке и она взаимодействует с магнитным полем постоянного магнита или электромагнита и поворачивается до тех пор, пока ее плоскость не станет параллельной вектору магнит ной индукции. Чтобы она могла нужно сменить направление силы тока в ней, вследствие чего поменяет направлению сила Ампера, действующая на рамку с током в магнитном поле. В двигателе этот процесс осуществляется с помощью двух неподвижных графитометаллических щеток и двух полуколец на валу, к которым подведены концы рамки.

    На рисунке 2.14-а показан момент, когда ток в якоре такого направления, что его полюса отталкиваются от одноименных полюсов статора. После поворачивания на некоторый угол якорь окажется в положении, когда разноименные полюса притягиваются (рис 2.14-6). Вследствие инерции якорь проходит это положение равновесия, а благодаря кольцам, которых касаются токоподводящие щетки (рис. 2.14-в), направление тока в якоре изменяется па противоположное и вращение якоря продолжается (см. рис. 2.14-а).

    Сила и закон Ампера - формулы и определение с примерами
    Pиc. 2.14 Схемы, которые объясняют действие коллекторного электродвигателя постоянного тока

    В промышленных образцах электродвигателей постоянного тока ротор имеет несколько рамок-обмоток. Поэтому и количество пар скользящих контактов в них больше: оно согласуется с количеством обмоток. В целом такое устройство называют коллектором. В новейших моделях двигателей постоянного тока роль коллектора выполняет специальное устройство с электронными приборами.

    Таким образом, действие силы Ампера нашло применение в различных технических устройствах: электроизмерительных приборах, электрических двигателях и т. п.

    Сила ампера

    Вы узнали, что магнитное поле действует на проводник с током с некоторой силой. А из курса физики 8 класса помните, что сила — это векторная физическая величина, поэтому, чтобы полностью определить силу, нужно уметь рассчитывать ее значение и определять направление. От чего зависит значение силы, с которой магнитное поле действует на проводник с током, как направлена эта сила и почему ее называют силой Ампера, вы узнаете из данного параграфа.

    Характеристика силы действующей на проводник с током

    Между полюсами подковообразного постоянного магнита подвесим на тонких и гибких проводах прямой алюминиевый проводник (рис. 4.1, а). Если через проводник пропустить ток, проводник отклонится от положения равновесия (рис. 4.1, б). Причина такого отклонения — сила, действующая на проводник с током со стороны магнитного поля. Доказал наличие этой силы и выяснил, от чего зависят ее значение и направление, А. Ампер. Именно потому эту силу называют силой Ампера.

    Сила и закон Ампера - формулы и определение с примерами

    Рис. 4.1. Опыт, демонстрирующий действие магнитного поля на алюминиевый проводник: при отсутствии тока магнитное поле на проводник не действует (а); если в проводнике течет ток, на проводник действует магнитное поле и проводник отклоняется (б)

    Сила Ампера — это сила, с которой магнитное поле действует на проводник с током.

    Сила Ампера прямо пропорциональна силе тока в проводнике и длине активной части проводника (то есть части, расположенной в магнитном поле). Сила Ампера увеличивается с увеличением индукции магнитного поля и зависит от того, под каким углом к линиям магнитной индукции расположен проводник.

    Значение силы Ампера Сила и закон Ампера - формулы и определение с примерами вычисляют по формуле:

    Сила и закон Ампера - формулы и определение с примерами

    где Сила и закон Ампера - формулы и определение с примерами — магнитная индукция магнитного поля; Сила и закон Ампера - формулы и определение с примерами — сила тока в проводнике; Сила и закон Ампера - формулы и определение с примерами — длина активной части проводника; Сила и закон Ампера - формулы и определение с примерами — угол между направлением вектора магнитной индукции и направлением тока в проводнике (рис. 4.2).

    Обратите внимание! Магнитное поле не будет действовать на проводник с током Сила и закон Ампера - формулы и определение с примерами если проводник расположен параллельно магнитным линиям поля Сила и закон Ампера - формулы и определение с примерами

    Сила и закон Ампера - формулы и определение с примерами

    Рис. 4.2. Угол Сила и закон Ампера - формулы и определение с примерами — это угол между направлением вектора магнитной индукции и направлением тока в проводнике

    Чтобы определить направление силы Ампера, используют правило левой руки:

    Если левую руку расположить так, чтобы линии магнитной индукции входили в ладонь, а четыре вытянутых пальца указывали направление тока в проводнике, то отогнутый на 90° большой палец укажет направление силы Ампера (рис. 4.3).

    Сила и закон Ампера - формулы и определение с примерами

    Рис. 4.3. Определение направления силы Ампера по правилу левой руки

    Формула для определения модуля магнитной индукции

    Если проводник расположен перпендикулярно линиям магнитной индукции Сила и закон Ампера - формулы и определение с примерами поле действует на проводник с максимальной силой:

    Сила и закон Ампера - формулы и определение с примерами

    Отсюда получаем формулу для определения модуля магнитной индукции:

    Сила и закон Ампера - формулы и определение с примерами

    Обратите внимание! Значение магнитной индукции не зависит ни от силы тока в проводнике, ни от длины проводника, а зависит только от свойств магнитного поля.

    Например, если уменьшить силу тока в проводнике, то уменьшится и сила Ампера, с которой магнитное поле действует на проводник, а вот значение магнитной индукции останется неизменным.

    В СИ единица магнитной индукции — тесла (Тл), единица силы — ньютон (Н), силы тока — ампер (А), длины — метр (м), поэтому:

    Сила и закон Ампера - формулы и определение с примерами

    1 Тл — это индукция такого однородного магнитного поля, которое действует с максимальной силой 1 Н на проводник длиной 1 м, в котором течет ток силой 1 А.

    • Заказать решение задач по физике

    Пример №3

    Докажите, что два параллельных проводника, в которых текут токи одного направления, притягиваются.

    Анализ физической проблемы. Около любого проводника с током существует магнитное поле, следовательно, каждый из двух проводников находится в магнитном поле другого. На первый проводник действует сила Ампера со стороны магнитного поля, созданного током во втором проводнике, и наоборот. Определив по правилу левой руки направления этих сил, выясним, как будут вести себя проводники.

    Решение

    Решая задачу, выполним пояснительные рисунки: изобразим проводники А и В, покажем направления тока в них и т. д.

    Выясним направление силы Ампера, которая действует на проводник А, находящийся в магнитном поле проводника В.

    1. С помощью правила буравчика найдем направление линий магнитной индукции магнитного поля, созданного проводником В (рис. 1, а). Выясняется, что вблизи проводника А магнитные линии направлены к нам (обозначено «•»).
    2. Воспользовавшись правилом левой руки, определим направление силы Ампера, действующей на проводник А со стороны магнитного поля проводника В (рис. 1, б).

    Сила и закон Ампера - формулы и определение с примерами

    Рис. 1

    3. Приходим к выводу: проводник А притягивается к проводнику В.

    Теперь выясним направление силы Ампера, которая действует на проводник В, находящийся в магнитном поле проводника А.

    1) Определим направление линий магнитной индукции магнитного поля, созданного проводником А (рис. 2, а). Выясняется, что вблизи проводника В магнитные линии направлены от нас (обозначено Сила и закон Ампера - формулы и определение с примерами

    2) Определим направление силы Ампера, действующей на проводник В (рис. 2, б).

    Сила и закон Ампера - формулы и определение с примерами

    Рис. 2

    3) Приходим к выводу: проводник В притягивается к проводнику А.

    Ответ: два параллельных проводника, в которых текут токи одного направления, притягиваются.

    Пример №4

    Прямой проводник (стержень) длиной 0,1 м и массой 40 г находится в горизонтальном однородном магнитном поле индукцией 0,5 Тл. Стержень расположен перпендикулярно магнитным линиям поля (рис. 3).

    Сила и закон Ампера - формулы и определение с примерами

    Рис. 3

    Ток какой силы и в каком направлении следует пропустить по стержню, чтобы стержень не давил на опору (завис в магнитном поле)?

    Анализ физической проблемы. Стержень не будет давить на опору, если сила Ампера уравновесит силу тяжести. Это произойдет при условиях: 1) сила Ампера будет направлена противоположно силе тяжести (то есть вертикально вверх); 2) значение силы Ампера будет равно значению силы тяжести: Сила и закон Ампера - формулы и определение с примерами

    Дано:

    Сила и закон Ампера - формулы и определение с примерами

    Найти:

    Сила и закон Ампера - формулы и определение с примерами

    Поиск математической модели, решение

    1. Определим направление тока. Для этого расположим левую руку так, чтобы линии магнитного поля входили в ладонь, а отогнутый на 90° большой палец был направлен вертикально вверх. Четыре вытянутых пальца укажут направление от нас. Следовательно, ток в проводнике нужно направить от нас.

    2. Учитываем, что Сила и закон Ампера - формулы и определение с примерами

    Сила и закон Ампера - формулы и определение с примерами где Сила и закон Ампера - формулы и определение с примерами

    Следовательно, Сила и закон Ампера - формулы и определение с примерами

    Из последнего выражения найдем силу тока: Сила и закон Ампера - формулы и определение с примерами

    Проверим единицу, найдем значение искомой величины.

    Вспомним: Сила и закон Ампера - формулы и определение с примерами

    Ответ: Сила и закон Ампера - формулы и определение с примерами от нас.

    Подводим итоги:

    Силу, с которой магнитное поле действует на проводник с током, называют силой Ампера. Значение силы Ампера находят по формуле: Сила и закон Ампера - формулы и определение с примерами где В — индукция магнитного поля; I — сила тока в проводнике; Сила и закон Ампера - формулы и определение с примерами — длина активной части проводника; Сила и закон Ампера - формулы и определение с примерами — угол между направлением вектора магнитной индукции и направлением тока в проводнике.

    Направление силы Ампера определяют по правилу левой руки: если левую руку расположить так, чтобы линии магнитной индукции входили в ладонь, а четыре вытянутых пальца указывали направление тока в проводнике, то отогнутый на 90° большой палец укажет направление силы Ампера.

    Магнитные свойства веществ и гипотеза Ампера

    Наверное, каждый из вас видел магниты и даже исследовал их свойства. Если поднести магнит к кучке мелких предметов, некоторые из них (гвоздики, кнопки, скрепки) притянутся к магниту, а некоторые (кусочки мела, медные и алюминиевые монетки, комочки земли) никак не отреагируют. Почему так? Действительно ли магнитное поле не оказывает никакого влияния на некоторые вещества? Именно об этом пойдет речь в параграфе.

    Действия электрического и магнитного полей на вещество

    Изучая в 8 классе электрические явления, вы узнали, что под влиянием внешнего электрического поля происходит перераспределение электрических зарядов внутри незаряженного тела (рис. 5.1). В результате в теле образуется собственное электрическое поле, направленное противоположно внешнему, и именно поэтому электрическое поле в веществе всегда ослабляется.

    Сила и закон Ампера - формулы и определение с примерами

    Рис. 5.1. В результате действия электрического поля отрицательно заряженной палочки ближняя к ней часть проводящей сферы приобретает положительный заряд

    Вещество изменяет и магнитное поле. Есть вещества, которые (как в случае с электрическим полем) ослабляют магнитное поле внутри себя. Такие вещества называют диамагнетиками. Многие вещества, наоборот, усиливают магнитное поле — это парамагнетики и ферромагнетики.

    Дело в том, что любое вещество, помещенное в магнитное поле, намагничивается, то есть создает собственное магнитное поле, магнитная индукция которого разная для разных веществ.

    Слабомагнитные вещества

    Вещества, которые намагничиваются, создавая слабое магнитное поле, магнитная индукция которого намного меньше магнитной индукции внешнего магнитного поля (то есть поля, вызвавшего намагничивание), называют слабомагнитными веществами. К таким веществам относятся диамагнетики и парамагнетики.

    Диамагнетики (от греч. dia — расхождение) намагничиваются, создавая слабое магнитное поле, направленное противоположно внешнему магнитному полю (рис. 5.2, а). Именно поэтому диамагнетики незначительно ослабляют внешнее магнитное поле: магнитная индукция магнитного поля внутри диамагнетика Сила и закон Ампера - формулы и определение с примерами немного меньше магнитной индукции внешнего магнитного поля Сила и закон Ампера - формулы и определение с примерами

    Сила и закон Ампера - формулы и определение с примерами

    Сила и закон Ампера - формулы и определение с примерами

    Рис. 5.2. Образцы из диамагнетика (а) и парамагнетика (б) во внешнем магнитном поле: красные линии — линии магнитного поля, созданного образцом; синие — магнитные линии внешнего магнитного поля; зеленые — линии результирующего магнитного поля

    Если диамагнетик поместить в магнитное поле, он будет выталкиваться из него (рис. 5.3).

    Сила и закон Ампера - формулы и определение с примерами

    Рис. 5.3. Пламя свечи выталкивается из магнитного поля, так как продукты сгорания — диамагнитные частицы

    К диамагнетикам относятся инертные газы (гелий, неон и др.), многие металлы (золото, медь, ртуть, серебро и др.), молекулярный азот, вода и т. д. Тело человека — диамагнетик, так как оно в среднем на 78 % состоит из воды.

    Парамагнетики (от греч. para — рядом) намагничиваются, создавая слабое магнитное поле, направленное в ту же сторону, что и внешнее магнитное поле (рис. 5.2, б). Парамагнетики незначительно усиливают внешнее поле: магнитная индукция магнитного поля внутри парамагнетика Сила и закон Ампера - формулы и определение с примерами немного больше магнитной индукции внешнего магнитного поля Сила и закон Ампера - формулы и определение с примерами

    Сила и закон Ампера - формулы и определение с примерами

    К парамагнетикам относятся кислород, платина, алюминий, щелочные и щелочноземельные металлы и другие вещества. Если парамагнитное вещество поместить в магнитное поле, то оно будет втягиваться в это поле.

    Ферромагнетики

    Если слабомагнитные вещества извлечь из магнитного поля, их намагниченность сразу исчезнет. Иначе происходит с сильномагнитными веществами — ферромагнетиками.

    Ферромагнетики (от лат. ferrum — железо) — вещества или материалы, которые остаются намагниченными и при отсутствии внешнего магнитного поля.

    Ферромагнетики намагничиваются, создавая сильное магнитное поле, направленное в ту же сторону, что и внешнее магнитное поле (рис. 5.4, 5.5, а). Если изготовленное из ферромагнетика тело поместить в магнитное поле, оно будет втягиваться в него (рис. 5.5, б).

    Сила и закон Ампера - формулы и определение с примерами

    Рис. 5.4. Железный гвоздь намагничивается в магнитном поле так, что конец гвоздя, расположенный вблизи северного полюса магнита, становится южным полюсом, поэтому гвоздь притягивается к магниту

    Сила и закон Ампера - формулы и определение с примерами

    Рис. 5.5. Ферромагнетики создают сильное магнитное поле, направленное в ту же сторону, что и внешнее магнитное поле (а); линии магнитной индукции как будто втягиваются в ферромагнитный образец (б)

    К ферромагнетикам относится небольшая группа веществ: железо, никель, кобальт, редкоземельные вещества и ряд сплавов. Ферромагнетики значительно усиливают внешнее магнитное поле: магнитная индукция магнитного поля внутри ферромагнетиков Сила и закон Ампера - формулы и определение с примерами в сотни и тысячи раз больше магнитной индукции внешнего магнитного поля Сила и закон Ампера - формулы и определение с примерами

    Сила и закон Ампера - формулы и определение с примерами

    Так, кобальт усиливает магнитное поле в 175 раз, никель — в 1120 раз, а трансформаторная сталь (на 96-98 % состоит из железа) — в 8000 раз.

    Ферромагнитные материалы условно делят на два типа. Материалы, которые после прекращения действия внешнего магнитного поля остаются намагниченными длительное время, называют магнитожесткими ферромагнетиками. Их применяют для изготовления постоянных магнитов. Ферромагнитные материалы, которые легко намагничиваются и быстро размагничиваются, называют магнитомягкими ферромагнетиками. Их применяют для изготовления сердечников электромагнитов, двигателей, трансформаторов, то есть устройств, которые во время работы постоянно перемагничиваются (о строении и принципе действия таких устройств вы узнаете позже).

    Обратите внимание! При достижении температуры Кюри (см. таблицу) ферромагнитные свойства магнитомягких и магнитожестких материалов исчезают — материалы становятся парамагнетиками.

    Температура Кюри для некоторых ферромагнетиков

    Вещество (или материал) Температура,°С
    Гадолиний +19
    Железо +770
    Кобальт +1127
    Неодимовый магнит NdFeB +320
    Никель +354

    Гипотеза Ампера

    Наблюдая действие проводника с током на магнитную стрелку (см. рис. 1.1) и выяснив, что катушки с током ведут себя как постоянные магниты (см. рис. 1.3), А. Ампер выдвинул гипотезу о магнитных свойствах веществ. Ампер предположил, что внутри веществ существует огромное количество незатухающих малых круговых токов и каждый из них, как маленькая катушка, является магнитиком. Постоянный магнит состоит из множества таких элементарных магнитиков, ориентированных в определенном направлении.

    Механизм намагничивания веществ Ампер объяснял так. Если тело не намагничено, круговые токи ориентированы беспорядочно (рис. 5.7, а). Внешнее магнитное поле пытается сориентировать эти токи так, чтобы направление магнитного поля каждого тока совпадало с направлением внешнего

    Сила и закон Ампера - формулы и определение с примерами

    Рис. 5.7. Механизм намагничивания тел согласно гипотезе Ампера: а — круговые токи ориентированы беспорядочно, тело не намагничено; б — круговые токи ориентированы в определенном направлении, тело намагничено

    магнитного поля (рис. 5.7, б). У некоторых веществ такая ориентация токов (намагничивание) остается и после прекращения действия внешнего магнитного поля. Таким образом, все магнитные явления Ампер объяснял взаимодействием движущихся заряженных частиц.

    Гипотеза Ампера послужила толчком к созданию теории магнетизма. На основе этой гипотезы были объяснены известные свойства ферромагнетиков, однако она не могла объяснить природу диа- и парамагнетизма, а также то, почему только небольшое количество веществ имеет ферромагнитные свойства. Современная теория магнетизма основана на законах квантовой механики и теории относительности А. Эйнштейна.

    Подводим итоги:

    Любое вещество, помещенное в магнитное поле, намагничивается, то есть создает собственное магнитное поле.

    Диамагнетики Парамагнетики Ферромагнетики
    Намагничиваются, создавая слабое магнитное поле, направленное противоположно внешнему магнитному полю Намагничиваются, создавая слабое магнитное поле, направленное в сторону внешнего магнитного поля Намагничиваются, создавая сильное магнитное поле, направленное в сторону внешнего магнитного поля; остаются намагниченными после прекращения действия внешнего магнитного поля
    Незначительно ослабляют внешнее магнитное поле, выталкиваются из него Незначительно усиливают внешнее магнитное поле, втягиваются в него Усиливают внешнее магнитное поле в сотни и тысячи раз, втягиваются в него
    Инертные газы, медь, золото, ртуть, серебро, азот, вода и др. Кислород, платина, алюминий, щелочные металлы и др. Кислород, платина, алюминий, щелочные металлы и др. Железо, никель, кобальт, редкоземельные вещества (например, неодим), ряд сплавов
    • Закон взаимодействия прямолинейных параллельных проводников с током
    • Сила Лоренца
    • Правило Буравчика в физике
    • Шунт и добавочное сопротивление
    • Работа по перемещению заряда в электростатическом поле
    • Закон Ома для однородного участка электрической цепи
    • Закон Ома для полной цепи
    • Закон Ома для цепи переменного тока с последовательным соединением сопротивлений

    Понравилась статья? Поделить с друзьями:
  • Кривой копчик как исправить
  • Как нужно найти человека по номеру телефона
  • Как найти спам в почте mail
  • Как найти платежеспособных клиентов
  • Как в камине исправить вычеты